Diễn Đàn MathScopeDiễn Đàn MathScope
  Diễn Đàn MathScope
Ghi Danh Hỏi/Ðáp Community Lịch

Go Back   Diễn Đàn MathScope > Sơ Cấp > Đại Số và Lượng Giác > Các Bài Toán Đã Được Giải

News & Announcements

Ngoài một số quy định đã được nêu trong phần Quy định của Ghi Danh , mọi người tranh thủ bỏ ra 5 phút để đọc thêm một số Quy định sau để khỏi bị treo nick ở MathScope nhé !

* Nội quy MathScope.Org

* Một số quy định chung !

* Quy định về việc viết bài trong diễn đàn MathScope

* Nếu bạn muốn gia nhập đội ngũ BQT thì vui lòng tham gia tại đây

* Những câu hỏi thường gặp

* Về việc viết bài trong Box Đại học và Sau đại học


Ðề tài đã khoá Gởi Ðề Tài Mới
 
Ðiều Chỉnh Xếp Bài
Old 23-08-2010, 11:13 PM   #46
asd257
+Thành Viên+
 
asd257's Avatar
 
Tham gia ngày: Aug 2010
Bài gởi: 25
Thanks: 22
Thanked 20 Times in 14 Posts
Icon10 Bất đẳng thức

Cho a,b,c >0
chứng minh:
$\frac{b+c}{2a^2+bc}+\frac{c+a}{2b^2+ca} +\frac{a+b}{2c^2+ab} \geq \frac{6}{a+b+c} $
[RIGHT][I][B]Nguồn: MathScope.ORG[/B][/I][/RIGHT]
 

thay đổi nội dung bởi: asd257, 24-08-2010 lúc 12:22 PM
asd257 is offline  
The Following User Says Thank You to asd257 For This Useful Post:
IMO 2010 (27-11-2010)
Old 23-08-2010, 11:46 PM   #47
leminhphuc
+Thành Viên+
 
Tham gia ngày: Dec 2009
Bài gởi: 3
Thanks: 3
Thanked 4 Times in 1 Post
Áp dụng bất đẳng thức Schwart ta có
$\sum_{cyc}\frac{b+c}{2a^2+bc} = \sum_{cyc}\frac{(b+c)^{2}}{\left (2a^2+bc \right )\left (b+c \right )} \geq \frac{\left (a+b+b+c+c+a \right )^2}{\sum_{cyc}\left (2a^2+bc \right )\left (b+c \right )} $

Lại có

$\sum_{cyc}\left (2a^2+bc \right )\left (b+c \right ) = 3\left (ab\left ( a+b \right ) + bc\left (b+c \right )+ ca\left (c+a \right ) \right ) \leq 3\left (a^3 + b^3 + c^3 + 3abc \right ) < 4\left ( a^3+b^3+c^3+3\left (a+b \right )\left ( b+c \right )\left ( c+a \right ) \right ) = 4\left ( a+b+c \right )^3 $

Từ 2 bất đẳng thức trên suy ra điều phải chứng minh.
Đẳng thức không xảy ra.


Mình cảm thấy bất đẳng thức này chưa đủ chặt. Các bạn thử làm chặt hơn xem có được không?????
[RIGHT][I][B]Nguồn: MathScope.ORG[/B][/I][/RIGHT]
 

thay đổi nội dung bởi: leminhphuc, 23-08-2010 lúc 11:51 PM
leminhphuc is offline  
The Following 4 Users Say Thank You to leminhphuc For This Useful Post:
asd257 (24-08-2010), h.vuong_pdl (24-08-2010), ha linh (02-02-2011), IMO 2010 (27-11-2010)
Old 24-08-2010, 04:08 AM   #48
huynhcongbang
Administrator

 
huynhcongbang's Avatar
 
Tham gia ngày: Feb 2009
Đến từ: Ho Chi Minh City
Bài gởi: 2,413
Thanks: 2,165
Thanked 4,188 Times in 1,381 Posts
Gửi tin nhắn qua Yahoo chát tới huynhcongbang
Trích:
Nguyên văn bởi asd257 View Post
Cho a,b,c > 0 , $a+b+c = \frac{1}{a}+\frac{1}{b}+\frac{1}{c} $
Chứng minh:
b) $2(a+b+c) \geq \sqrt{a^2+3} +\sqrt{b^2+3} + \sqrt{c^2+3 $
Mình giải thử câu b/
Điều kiện đã cho tương đương với:
$a+b+c = \frac{1}{a}+\frac{1}{b}+\frac{1}{c}\Leftrightarrow \frac{a^2-1}{a}+\frac{b^2-1}{b}+\frac{c^2-1}{c} =0 $.
BĐT cần chứng minh tương đương với:
$2(a+b+c) \geq \sqrt{a^2+3} +\sqrt{b^2+3} + \sqrt{c^2+3} \\\Leftrightarrow \frac{a^2-1}{2a+\sqrt{a^2+3}}+\frac{b^2-1}{2b+\sqrt{b^2+3}}+\frac{c^2-1}{2c+\sqrt{c^2+3}}\ge 0\\\Leftrightarrow \frac{\frac{a^2-1}{a}}{2+\sqrt{1+\frac{3}{a^2}}}+\frac{\frac{b^2-1}{b}}{2+\sqrt{1+\frac{3}{b^2}}}+\frac{\frac{c^2-1}{c}}{2+\sqrt{1+\frac{3}{c^2}}} \ge 0 $.
Giả sử $a \ge b \ge c $ thì:
$\frac{a^2-1}{a} \ge \frac{b^2-1}{b} \ge \frac{c^2-1}{c} $
$\frac{1}{2+\sqrt{1+\frac{3}{a^2}}} \ge \frac{1}{2+\sqrt{1+\frac{3}{b^2}}} \ge \frac{1}{2+\sqrt{1+\frac{3}{c^2}}} $.
Theo BĐT Chebyshev, ta có:
$VT \ge \frac{1}{3}(\sum \frac{a^2-1}{a})(\sum \frac{1}{2+\sqrt{1+\frac{3}{a^2}}})=0 $.
Ta có đpcm.
Đẳng thức xảy ra khi $a=b=c=1 $.

Mình nghĩ câu c/ cũng có thể làm tương tự.
[RIGHT][I][B]Nguồn: MathScope.ORG[/B][/I][/RIGHT]
 

thay đổi nội dung bởi: huynhcongbang, 24-08-2010 lúc 04:11 AM
huynhcongbang is offline  
The Following 4 Users Say Thank You to huynhcongbang For This Useful Post:
asd257 (24-08-2010), boheoga9999 (24-08-2010), ha linh (02-02-2011), IMO 2010 (27-11-2010)
Old 24-08-2010, 08:00 AM   #49
h.vuong_pdl
+Thành Viên+
 
Tham gia ngày: Jul 2010
Bài gởi: 56
Thanks: 18
Thanked 32 Times in 20 Posts
Hỏi bài BDT ???

Cho x,y,z là các số thực thỏa mãn $x+y+z=xy+yz+zx $.
Tìm GTNN của $M = \sum{\frac{xz}{x^2+1}} $
[RIGHT][I][B]Nguồn: MathScope.ORG[/B][/I][/RIGHT]
 
h.vuong_pdl is offline  
The Following User Says Thank You to h.vuong_pdl For This Useful Post:
IMO 2010 (27-11-2010)
Old 24-08-2010, 08:54 AM   #50
hoangduyenkhtn
+Thành Viên+
 
Tham gia ngày: Dec 2008
Bài gởi: 71
Thanks: 56
Thanked 57 Times in 36 Posts
Trao đổi về một bất đẳng thức đẹp

Cho ba số thực dương a,b,c thỏa mãn a+b+c=3.CMR
$\frac{ab}{\sqrt{c^2+3}}+\frac{bc}{\sqrt{a^2+3}}+ \frac{ca}{\sqrt{b^2+3}} \le \frac{3}{2} $.
Mong rằng các bạn sẽ đóng góp lời giải cho bài toán này.Chúc toàn thể diễn đàn ngày mới học tập và làm việc hiệu quả.
[RIGHT][I][B]Nguồn: MathScope.ORG[/B][/I][/RIGHT]
 

thay đổi nội dung bởi: novae, 24-08-2010 lúc 11:48 AM
hoangduyenkhtn is offline  
The Following 3 Users Say Thank You to hoangduyenkhtn For This Useful Post:
hoaian1294 (05-04-2011), IMO 2010 (27-11-2010), maihoangquyet (02-06-2011)
Old 24-08-2010, 10:50 AM   #51
h.vuong_pdl
+Thành Viên+
 
Tham gia ngày: Jul 2010
Bài gởi: 56
Thanks: 18
Thanked 32 Times in 20 Posts
Bài 1) chú ý gt => $abc(a+b+c) = ab+bc+ca $
Nhân thêm 2 vế của BDT cần Cm thì ta có: $5(a+b+c)^2 \ge 7(a+b+c) + 8(ab+bc+ca) $
dễ thấy $3(ab+bc+ca) \le (a+b+c)^2 $ nên chỉ cần Cm:
$7(a+b+c)^2 \ge 21(a+b+c) hay a+b+c \ge 3 $.
Giả sử ngược lại $a+b+c < 3 $ thì $abc < 1 $
chú ý gt $3\sqrt[3]{a^2b^2c^2} \le ab+bc+ca = abc(a+b+c) < 3abc => abc >1 $ => mâu thuẫn ==> đpcm !1111111
[RIGHT][I][B]Nguồn: MathScope.ORG[/B][/I][/RIGHT]
 
h.vuong_pdl is offline  
The Following 4 Users Say Thank You to h.vuong_pdl For This Useful Post:
asd257 (24-08-2010), boheoga9999 (24-08-2010), ha linh (02-02-2011), IMO 2010 (27-11-2010)
Old 24-08-2010, 11:07 AM   #52
h.vuong_pdl
+Thành Viên+
 
Tham gia ngày: Jul 2010
Bài gởi: 56
Thanks: 18
Thanked 32 Times in 20 Posts
@leminhphuc: cái này có lẽ là do lỗi tác giả đánh sai thôi, phải là 6/(a+b+c) cơ ==> nó khá mạnh đó. khí giải bạn cần chú ý những BDT đẹp như trên thì dt xảy ra khi a=b=c ==> cứ thế mak sửa chỉnh đề lại đôi chút để giải

P/s: nhưng dù sao cũng .........
------------------------------
SoS nè. phân tích cơ sở:
$S_c = (2c^2+ab)((a+b)^2 + ab - bc - ca) $
$S_b, S_a $ tương tự thôi ???/
Có ai thử Cm tiếp không ???????????
------------------------------
SoS nè. phân tích cơ sở:
$S_c = (2c^2+ab)((a+b)^2 + ab - bc - ca) $
$S_b, S_a $ tương tự thôi ???/
Có ai thử Cm tiếp không ???????????
[RIGHT][I][B]Nguồn: MathScope.ORG[/B][/I][/RIGHT]
 

thay đổi nội dung bởi: h.vuong_pdl, 24-08-2010 lúc 11:21 AM Lý do: Tự động gộp bài
h.vuong_pdl is offline  
The Following 2 Users Say Thank You to h.vuong_pdl For This Useful Post:
asd257 (24-08-2010), IMO 2010 (27-11-2010)
Old 24-08-2010, 11:49 AM   #53
sonhadhsp
+Thành Viên+
 
Tham gia ngày: Aug 2010
Đến từ: Giáo viên Trường THPT Chuyên Đại học Sư phạm Hà Nội
Bài gởi: 107
Thanks: 3
Thanked 152 Times in 63 Posts
Trích:
Nguyên văn bởi h.vuong_pdl View Post
Cho ba số thực dương a,b,c thỏa mãn a+b+c=3.CMR
$\frac{ab}{\sqrt{c^2+3}}+\frac{bc}{\sqrt{a^2+3}}+\f rac{ca}{\sqrt{b^2+3}} \le \frac{3}{2} $.
$\ ab+bc+ca \le 3 $
$\frac{ab}{\sqrt{c^2+3}}+\frac{bc}{\sqrt{a^2+3}}+ \frac{ca}{\sqrt{b^2+3}} \le \frac{ab}{\sqrt{c^2+ab+bc+ca}}+\frac{bc}{\sqrt{a^2 +ab+bc+ca}}+\frac{ca}{\sqrt{b^2+ab+bc+ca}} $
$\le \frac{ab}{\sqrt{(c+a)(c+b)}}+\frac{bc}{\sqrt{(a+b) (a+c)}}+\frac{ca}{\sqrt{(b+c)(b+a)}} $
$\le \frac{1}{2}(\frac{ab}{c+a}+\frac{ab}{c+b}+\frac{bc }{c+a}+\frac{bc}{a+b}+\frac{ca}{b+a}+\frac{ca}{c+b }) =\frac{1}{2}(a+b+c)=\frac{3}{2} $
[RIGHT][I][B]Nguồn: MathScope.ORG[/B][/I][/RIGHT]
 

thay đổi nội dung bởi: novae, 24-08-2010 lúc 11:54 AM Lý do: sửa Latex
sonhadhsp is offline  
The Following 3 Users Say Thank You to sonhadhsp For This Useful Post:
huynhcongbang (24-08-2010), IMO 2010 (27-11-2010), tinnguyen (24-08-2010)
Old 24-08-2010, 12:20 PM   #54
asd257
+Thành Viên+
 
asd257's Avatar
 
Tham gia ngày: Aug 2010
Bài gởi: 25
Thanks: 22
Thanked 20 Times in 14 Posts
Trích:
Nguyên văn bởi asd257 View Post
Cho a,b,c >0
chứng minh:
$\frac{b+c}{2a^2+bc}+\frac{c+a}{2b^2+ca} +\frac{a+b}{2c^2+ab} \geq \frac{1}{a+b+c} $
sr các bạn Mình type nhầm đề mà chả chú ý
bên vế phải là
$\frac{6}{a+b+c} $
[RIGHT][I][B]Nguồn: MathScope.ORG[/B][/I][/RIGHT]
 
asd257 is offline  
The Following User Says Thank You to asd257 For This Useful Post:
IMO 2010 (27-11-2010)
Old 24-08-2010, 12:36 PM   #55
asd257
+Thành Viên+
 
asd257's Avatar
 
Tham gia ngày: Aug 2010
Bài gởi: 25
Thanks: 22
Thanked 20 Times in 14 Posts
Icon6 Không phải tất cả đều dương (bdt)

Cho x,y,z là các số thực không phải tất cả đều dương
chứng minh:
$\frac{16}{9}(x^2-x+1)(y^2-y+1)(z^2-z+1) \geq (xyz)^2-xyz+1 $
[RIGHT][I][B]Nguồn: MathScope.ORG[/B][/I][/RIGHT]
 
asd257 is offline  
The Following User Says Thank You to asd257 For This Useful Post:
IMO 2010 (27-11-2010)
Old 24-08-2010, 01:18 PM   #56
353535
Banned
 
Tham gia ngày: Jul 2010
Đến từ: LVT_NB
Bài gởi: 134
Thanks: 3
Thanked 61 Times in 38 Posts
Gửi tin nhắn qua Yahoo chát tới 353535
Trích:
Nguyên văn bởi asd257 View Post
Cho x,y,z là các số thực không phải tất cả đều dương
chứng minh:
$\frac{16}{9}(x^2-x+1)(y^2-y+1)(z^2-z+1) \geq (xyz)^2-xyz+1 $
BĐT sai !
VD:$x=y=z=\frac{1}{2} $
[RIGHT][I][B]Nguồn: MathScope.ORG[/B][/I][/RIGHT]
 
353535 is offline  
The Following User Says Thank You to 353535 For This Useful Post:
IMO 2010 (27-11-2010)
Old 24-08-2010, 03:04 PM   #57
asd257
+Thành Viên+
 
asd257's Avatar
 
Tham gia ngày: Aug 2010
Bài gởi: 25
Thanks: 22
Thanked 20 Times in 14 Posts
Trích:
Nguyên văn bởi 353535 View Post
BĐT sai !
VD:$x=y=z=\frac{1}{2} $
không phải tất cả đều dương mà bạn , vd của bạn là dương hết òy
[RIGHT][I][B]Nguồn: MathScope.ORG[/B][/I][/RIGHT]
 
asd257 is offline  
The Following User Says Thank You to asd257 For This Useful Post:
IMO 2010 (27-11-2010)
Old 24-08-2010, 03:16 PM   #58
hoangduyenkhtn
+Thành Viên+
 
Tham gia ngày: Dec 2008
Bài gởi: 71
Thanks: 56
Thanked 57 Times in 36 Posts
Cách giải của bạn giống mình.Bài này xuất hiện trong một quyển sách mình thấy lời giải của họ hơi phức tạp,Mình thấy dùng AM-GM cho bài này cũng hay.có bạn nào có cách giải hay hơn thì post lên trao đổi nha
[RIGHT][I][B]Nguồn: MathScope.ORG[/B][/I][/RIGHT]
 
hoangduyenkhtn is offline  
The Following User Says Thank You to hoangduyenkhtn For This Useful Post:
IMO 2010 (27-11-2010)
Old 24-08-2010, 03:18 PM   #59
legend
+Thành Viên+
 
legend's Avatar
 
Tham gia ngày: Mar 2010
Bài gởi: 57
Thanks: 16
Thanked 15 Times in 13 Posts
Icon1 Một bài bất đẳng thức.

Cho a, b, c là các số thực thuộc [-1;1] thỏa: $a^{2}+b^{2}+c^{2}\le 2abc+1 $.
CMR:
$a^{2n}+b^{2n}+c^{2n}\le 2a^{n}b^{n}c^{n}+1 $
[RIGHT][I][B]Nguồn: MathScope.ORG[/B][/I][/RIGHT]
 

thay đổi nội dung bởi: novae, 24-08-2010 lúc 05:50 PM
legend is offline  
The Following User Says Thank You to legend For This Useful Post:
IMO 2010 (27-11-2010)
Old 24-08-2010, 03:21 PM   #60
xuanquan
+Thành Viên+
 
Tham gia ngày: Mar 2010
Bài gởi: 44
Thanks: 64
Thanked 26 Times in 12 Posts
Bài này chuẩn hóa a+b+c=6 sau đó chú ý thêm là 4ab<=(a+b)2=(6-c)2 biến đổi tí sau đó dùng "pp tiếp tuyến" hoặc là U.C.T đều ra cả...
[RIGHT][I][B]Nguồn: MathScope.ORG[/B][/I][/RIGHT]
 
xuanquan is offline  
The Following 2 Users Say Thank You to xuanquan For This Useful Post:
asd257 (24-08-2010), IMO 2010 (27-11-2010)
Ðề tài đã khoá Gởi Ðề Tài Mới

Bookmarks

Tags
bất đẳng thức


Quuyền Hạn Của Bạn
You may not post new threads
You may not post replies
You may not post attachments
You may not edit your posts

BB code is Mở
Smilies đang Mở
[IMG] đang Mở
HTML đang Tắt

Chuyển đến


Múi giờ GMT. Hiện tại là 03:46 AM.


Powered by: vBulletin Copyright ©2000-2024, Jelsoft Enterprises Ltd.
Inactive Reminders By mathscope.org
[page compression: 104.72 k/121.54 k (13.84%)]